4
$\begingroup$

I'm reading about Friedman's statistic from the 3rd edition of Nonparametric Statistical Methods (Hollander, Wolfe and Chicken). On page 302 they have an exercise problem:

  1. Show directly, or illustrate by means of an example, that the maximum value of $S$ is $S_{\text{max}} = n ( k − 1 )$ . For what configuration is this maximum achieved?

They define $S$ as follows:

$\frac{12}{nk(k+1)}\sum_{j=1}^k R_j^2 - 3n(k+1)$,

where $R_j$ is the sum of group $j$ ranks, there are $k$ groups and $n$ blocks. Looking at the definition of $S$, I need to maximize $\sum R_j^2$. My uneducated guess is, that the maximum is achieved when the ranks of each column sum to most extreme values. But doing the algebra with groups $Rj$ having only rank $j$ $k$ times or distributing the ranks so that they start from group one having ranks from 1 to $n$ and continue in order to group two up to group $k$, which has ranks from $(k-1)n+1$ to $kn$ does not give me $n(k-1)$. So there must be some statistical expertise at play in the result? Could someone clarify the idea of how to maximize $S$?

They tell that with a large number of blocks, $S$ follows "asymptotic chi-squared distribution with $(k-1)$ degrees of freedom." What does that mean and is there a connection to the proposed maximum value?

$\endgroup$
1
  • 1
    $\begingroup$ It's & Chicken, not $. $\endgroup$ Commented Dec 2, 2024 at 19:48

1 Answer 1

4
$\begingroup$

The general form of the Friedman statistic using the ranks $R_1,...,R_k$ is:

$$Q(\mathbf{R}) = \frac{12n}{k(k+1)} \sum_{i=1}^k \bigg( R_i - \frac{k+1}{2} \bigg)^2.$$

The statistic is maximised when the ranks are $1,...,n$ (in any order), which gives the sums:

$$\begin{align} \sum_{i=1}^k R_i &= \frac{k(k+1)}{2}, \\[6pt] \sum_{i=1}^k R_i^2 &= \frac{k(k+1)(2k+1)}{6}. \\[6pt] \end{align}$$

This gives the maximised test statistic:

$$\begin{align} Q(\mathbf{R}) &= \frac{12n}{k(k+1)} \sum_{i=1}^k \bigg( R_i - \frac{k+1}{2} \bigg)^2 \\[6pt] &= \frac{12n}{k(k+1)} \bigg[ \sum_{i=1}^k R_i^2 - (k+1) \sum_{i=1}^k R_i + \frac{k(k+1)^2}{4} \bigg] \\[6pt] &= \frac{12n}{k(k+1)} \bigg[ \frac{k(k+1)(2k+1)}{6} - (k+1) \frac{k(k+1)}{2} + \frac{k(k+1)^2}{4} \bigg] \\[6pt] &= \frac{12n}{k(k+1)} \bigg[ \frac{k(k+1)(2k+1)}{6} - \frac{k(k+1)^2}{2} + \frac{k(k+1)^2}{4} \bigg] \\[6pt] &= \frac{n}{k} \bigg[ 2k(2k+1) - 6k(k+1) + 3k(k+1) \bigg] \\[6pt] &= \frac{n}{k} \bigg[ 4k^2 + 2k - 6k^2 - 6k + 3k^2 + 3k \bigg] \\[8pt] &= \frac{n}{k} (k^2 - k) \\[14pt] &= n(k-1). \\[6pt] \end{align}$$

$\endgroup$

Your Answer

By clicking “Post Your Answer”, you agree to our terms of service and acknowledge you have read our privacy policy.

Start asking to get answers

Find the answer to your question by asking.

Ask question

Explore related questions

See similar questions with these tags.